0 Daumen
1,1k Aufrufe

:)

Ich bin gaaanz am Anfang von Analysis 1 und hab nun die ersten Aufgaben zu Reihen gemacht.

Es wäre super wenn du kurz drüberschauen könntest, weil ich mir extrem unsicher bin ob das überhaupt passt - da ich alles mit dem Quotientenkriterium gelöst hat und wir natürlich noch viel mehr in der Vorlesung behandelt haben. Nun zur Aufgabe:

Entscheiden Sie, ob die folgenden Reihen konvergieren, absolut konvergieren oder divergieren (gegebenenfalls in Abhängigkeit von a ≥ 1). Begründen Sie Ihre Entscheidung.


screen.png

Text erkannt:

a) \( \sum \limits_{n=0}^{\infty} \frac{n}{(n+1) \cdot(n+2)} \)
\( a_{n}=\frac{11}{(n+1) \cdot(n+2)} \)
\( a_{n+1}=\frac{n+1}{(n+2) \cdot(n+3)} \)
Quotientenkitenum:
$$ \left|\frac{\frac{n+1}{(n+2) \cdot(n+3)}}{\frac{n}{(n+1) \cdot(n+2)}}\right|=\frac{n+1}{(n+2) \cdot(n+3)} \cdot \frac{(n+1) \cdot(2+2)}{n} $$
$$ \begin{array}{l} =\frac{(n+1)^{2}}{n^{2}+3 n}=\frac{n^{2}+2 n+1}{n^{2}+3 n} \\ =\frac{n \cdot\left(2 n+\frac{1}{n}\right)}{n \cdot(n+3)}=\frac{2+\frac{1}{n}}{n+3} \end{array} $$ \( \Delta 0<1 \Rightarrow \) konverg b) \( \sum \limits_{n=0}^{\infty} \frac{(2 i)^{n}}{n^{2}} \quad a_{n}=\frac{(2 i)^{n}}{n^{2}} \quad a_{n+1}=\frac{(2 i)^{n+1}}{(n+1)^{2}} \) Quotientenkiteinm \( \begin{aligned}\left(\frac{\frac{(2 i)^{n+1}}{(n+1)^{2}}}{\frac{(2 i)^{n}}{n^{2}}}\right.&=\frac{(2 i)^{n+1}}{(n+1)^{2}} \cdot \frac{n^{2}}{(2 i)^{n}} \\ &=\frac{(2 i)^{n} \cdot(2 i)}{n^{2}+2 n+1} \cdot \frac{n^{2}}{(i)^{2}} \end{aligned} \) \( = \) $$ =\frac{2 i \cdot n^{2}}{n^{2}+2 n+1}=\frac{x \cdot\left(\frac{2 i}{n} \cdot n\right)}{x \cdot\left(n+2+\frac{1}{r}\right)} $$ \( E \) $$ =\frac{21}{n+2+\frac{1}{n}} \frac{1}{n \rightarrow \infty} \underbrace{0}<1 \Rightarrow \begin{array}{c} n \cdot\left(n+2+\frac{n}{n}\right) \\ \text { anvesent } \end{array} $$


Das sind zumindest die beiden ersten Teilaufgabe. Wäre gut zu wissen ob es soweit stimmt, bevor ich weiter machen und dann alles falsch ist  

!!




Avatar von

1 Antwort

0 Daumen
 
Beste Antwort

n^2 + 2n + 1 = n (n + 2 + 1/n). Fehler beim ausklammern. Kürze aber besser gleich mit n^2. Danach ist aber auch nochmal ein Fehler:

2n + 1/n ist nicht 2+1/n. Dir ist da ein n verloren gegangen.


Bei der ersten Reihe wird das Quotientenkriterium nichts bringen. Vergleiche mit der harmonischen Reihe. Die könntet ihr in der VL schon behandelt haben.

Zur zweiten:


2i n^2 / ( n^2 + 2n + 1) ist auch nicht 2i / ( n + 2 +1/n) auch hier geht dir 1 n flöten.

Kürze auch hier mit n^2

Allgemein darfst du hier wegen dem i auch die Beträge nicht weglassen. Wenn du das korrigierst solltest du mit dem Quotientenkriterium die Divergenz nachweisen können.

Achte auch auf die Grenzen deiner Reihen, die zweite kann schlecht bei n=0 beginnen. Du teilst immerhin durch n^2.

Um die Bedeutung von "in Abhängigkeit von a" zu verstehen müsstest du uns auch den Rest der Aufgabe zeigen. Vermutlich taucht bei einer weiteren Reihe ein a in den Summanden auf. Dann hängt die Konvergenz natürlich vom Wert von a ab.

Avatar von 6,0 k

Okay Vielen Dank!! Wie ich bei der ersten Aufgabe mit der harmonischen Reihe weiterkomme ist mir allerdings noch nicht ganz klar. Schließlich steht der Laufindex auch im Zähler, ich dachte da darf nur eine Zahl stehen.

Achja das hab ich vergessen zu sagen dass in keiner Teilaufgabe ein "a" vorkommt. Deswegen  verwirrte mich dieses "in Abhängigkeit von a" auch so arg... naja

n/((n+1)(n+2)) = n / ( n^2 + 3n + 2 ) = 1 / ( n + 3 + 2/n ) für n > 0

Jetzt gilt ja 6n ≥ n + 3 + 2/n also 1/(6n) ≤ 1 / ( n + 3 + 2/n), aber damit kannst du jetzt doch abschätzen:

$$\sum_n \frac{1}{6n}=\frac{1}{6} \sum_n \frac{1}{n} = \frac{1}{6} \infty =\infty $$ und

$$ \infty = \sum_n \frac{1}{6n} \le \sum_n \frac{1}{n + 3 + 2/n} $$

Die Reihe divergiert also.

Okayyy macht soweit Sinn, danke dir! Kleine Frage zum Verständnis: Wie kommst du auf die 6n (6n ≥ n + 3 + 2/n), warum ist mir aber klar. LG!

Ach und hast du damit auch Gebrauch vom Minorantenkriterium gemacht oder?

Ich habe ein a mit

a*n ≥ n+3+2/n

gesucht um mit der harmonischen Reihe abschätzen zu können

n=1 eingesetzt: a≥6.

a=6 probiert, passte.

Edit: Ja, ich glaube das nennt man Minorantenkriterium.

Super, logisch! Nun bin ich dabei diese Reihe zu bestimmen: Sum(n=1 => infinity) of (n^2) + n + 5) / ((3n^4)-4n+5 . Sieht hässlich aus, ist es auch. Sehe da drin keine bekannte Reihe, Quotientenkriterium hat zu nichts geführt und Wurzelkriterium macht auch wenig Sinn. Hast du zufällig spontan ne Idee?

Im Zähler steht als höchste Potenz n², im Nenner 3n4. Der Quotient davon geht gegen 
1/(3n²), und die Reihe 1/n² konvergiert bekanntlich. Man sollte also  1/n² oder ein Vielfaches davon als konvergente Majorante nutzen.

Dazu müssen wir den Bruch (n² + n + 5) / (3n4-4n+5) vergrößern, und trotz des Vergrößerns muss noch Konvergenz vorliegen.

Im Zuge des Vergrößerns muss der Bruch so vereinfacht werden, dass das störende Beiwerk n+5 im Zähler und -4n+5 im Nenner verschwindet.

Zielstellung erkannt?

Ein Bruch wird vergrößert, indem man

- den Zähler vergrößert...

Offensichtlich gilt ab n=3 die Ungleichung n²>n+5 und somit n²+n²>n²+n+5.

Der Zähler wird also vergrößert auf 2n².

Ein Bruch wird auch vergrößert, indem man (zusätzlich noch)
- den Nenner verkleinert...

Für alle n gilt  n4>4n-5. Somit ist 3n4-n4 =2n4<3n4-(4n-5)=3n4-4n+5).

Damit gilt ab n=3

(n² + n + 5) / (3n4-4n+5) <(2n²)/(2n4)=1/n².

Letzteres konvergiert, ersteres dann erst recht.

Da würde ich erstmal mit \( n^2 \) kürzen

$$ \frac{n^2+n+5}{3n^4 - 4n +5} = \frac{1 + \frac{1}{n} + \frac{5}{n^2}}{3n^2 - \frac{4}{n} + \frac{5}{n^2}} $$

für große n ist das doch jetzt ungefähr in den Größenordnung von \( \frac{1}{n^2} \)

Ist dir die Reihe $$ \sum_{n=1}^\infty \frac{1}{n^2} $$ bekannt? Diese konvergiert (als kleiner "fun"fact: der Grenzwert der Reihe ist \( \frac{\pi^2}{6} \), den brauche wir aber im folgenden nicht)

Ich vermute deshalb, dass die Reihe ebenfalls konvergiert. für große n sind die Summanden nämlich ähnlich groß und was für kleine n passiert ist für das Konvergenzverhalten (für den Grenzwert aber nicht!) total egal.

Jetzt würde ich ein a suchen, s.d.

$$ \left| \frac{1 + \frac{1}{n} + \frac{5}{n^2}}{3n^2 - \frac{4}{n} + \frac{5}{n^2}} \right| \le \frac{a}{ n^2} $$

Und dann das Majorantenkriterium verwenden.

a = 42 passt bestimmt (geraten und ist schließlich die Antwort auf Alles).

Die Summanden sind auch alle nicht-negativ also:

$$ 0 \le \sum_{n=1}^\infty \frac{n^2+n+5}{3n^4 - 4n +5} \le \sum_{n=1}^\infty \frac{42}{n^2}  < \infty $$

Die Reihe konvergiert somit (sogar absolut).

Vielen Dank, alles verstanden! Gibt es eine Vorgehensweise, um möglichst schnell und sicher auf die 42 zu kommen? n = 1 einsetzen und gucken ob das a passt, falls nicht n = 2 einsetzen, dann n = 3 einsetzen usw wäre doch nicht sonderlich effizient oder?

Ein anderes Problem?

Stell deine Frage

Willkommen bei der Mathelounge! Stell deine Frage einfach und kostenlos

x
Made by a lovely community